- Wed Jul 13, 2016 1:40 pm
#27103
Complete Question Explanation
(The complete setup for this game can be found here: lsat/viewtopic.php?t=11371)
The correct answer choice is (A)
The final setup indicates that both S and U repair R, and that S repairs no other machine. Thus, answer choice (A) is correct.
Answer choice (B) is incorrect because it does not have to be true that X and Y share exactly one type of machine. They could have no machines in common, or one or two machines in common. Note that any answer choice that specifies an “exact” number of machines and that includes X will be suspect since X has so many options.
Answer choice (C) is incorrect because it does not have to be true that W and X share exactly one type of machine. They could have no machines in common, or one or two machines in common. Aside from the fact that this answer choice contains X and specifies an “exact” number of machines, it also contains W, which is also suspect in an “exact” question since W is also uncertain.
Answer choice (D) is incorrect because although this answer choice could be true, it does not have to be true (W could repair only one type of machine).
Note that the wording on the last two answer choices switches to “more than one” from the “exact” specification in answer choices (A), (B), and (C).
Answer choice (E) is incorrect because although this answer choice could be true, it does not have to be true (W could repair only one type of machine).
(The complete setup for this game can be found here: lsat/viewtopic.php?t=11371)
The correct answer choice is (A)
The final setup indicates that both S and U repair R, and that S repairs no other machine. Thus, answer choice (A) is correct.
Answer choice (B) is incorrect because it does not have to be true that X and Y share exactly one type of machine. They could have no machines in common, or one or two machines in common. Note that any answer choice that specifies an “exact” number of machines and that includes X will be suspect since X has so many options.
Answer choice (C) is incorrect because it does not have to be true that W and X share exactly one type of machine. They could have no machines in common, or one or two machines in common. Aside from the fact that this answer choice contains X and specifies an “exact” number of machines, it also contains W, which is also suspect in an “exact” question since W is also uncertain.
Answer choice (D) is incorrect because although this answer choice could be true, it does not have to be true (W could repair only one type of machine).
Note that the wording on the last two answer choices switches to “more than one” from the “exact” specification in answer choices (A), (B), and (C).
Answer choice (E) is incorrect because although this answer choice could be true, it does not have to be true (W could repair only one type of machine).